You are on page 1of 43

People Who Did Exceptionally Well on the LSAT

Chai_166
I haven't posted here before, but this thread was very helpful to me as I prepared for the LSAT so I wanted to
share my experience.

1) What score did you get?


179

2) What books did you use? (Kaplan, Powerscore LRB, Powerscore LGB, etc)
Princeton Review: I used this when I first started prep--good for a quick overview of the LSAT, but I think most
of their methods were more detrimental than helpful.
Powerscore Bibles! If you want to get a score above 170 I highly recommend these. I did a couple weeks of
prep before I started to go through them.
All available Preptests (but I didn't end up using 1-10 very much)

3) What prep courses did you take (if any)? Full length, weekend?
None. Thought about taking one, but decided to use the money I had to get all the prep tests, and the powerscore
bibles. In my area the only available course was Kaplan, so I might have chosen different if Testmasters was
near.

4) How long did you study for, and under what conditions? (during school, during the summer, etc)
2 months, while working full time

5) How many preptests did you do?


Hmmm.. I think I did around 40. Many have said this, but preptest are really the key to the LSAT. Make sure
you make a schedule for the prep tests so you are doing the newest ones leading up to the test. I started taking
the tests untimed, and then moved to timed tests. I also would do sections if I didn't have enough time for a
whole test. Other people have said this before, but make sure that you take at least some tests in distracting
situations. I took a couple at the book store. I also had a friend time me on couple of times. Having
someone else there made my adrenalin rush a bit, so simulated test day a little better.

6) What would you change if you were to do it again?


I wouldn't change much--I'm very happy with my score. I might skip the PR Book.

7) Any other misc comments/suggestions.


The week leading up to the test i took it pretty easy with LSAT prep--maybe did two tests. The day before the
test I took the day off work and just relaxed. The day of the test I got up around six and went out to
breakfast. I did a few problems to warm up, and then got to the test center in plenty of time. I really think
this made a difference on test day.

Although it might have been overkill, I kept a careful record of each preptest I took. I would record my score
and the number wrong in each section. I would then focus more on the section/question type I was doing
poorly on (do an untimed section from an older test, or review the specific section in the Powerscore Bible). It
was good to see what areas I needed help in, but it also felt good to see my improvement over time.

I also always tried to understand why I got questions wrong. If I messed up a game, I would erase
everything and do it again untimed. If I messed up a LR question I would cut it out and write out why I got the
question wrong. I kept a bunch of LR questions I had gotten wrong and would periodically review them. It's
important that you don't just do preptests over and over again. You have to understand why you got questions
wrong and make adjustments so you are always improving.

bcjets212

1) What score did you get?


first diagnostic, 153. then got 171 on June 08 and 180 on Oct 08

2) What books did you use? (Kaplan, Powerscore LRB, Powerscore LGB, etc)
Downloaded all the tests from each book

3) What prep courses did you take (if any)? Full length, weekend?
had a private tutor for a couple months before june

4) How long did you study for, and under what conditions? (during school, during the summer, etc)
i would study for 4-5 hours a day, 5 days a week leading up to june (was taking a practice test every day and
going back over it after). I would mix up the conditions that i took the practice tests under, a lot of times
intentionally putting myself in tough/loud environments for practice

5) How many preptests did you do?


40+

6) What would you change if you were to do it again?


not much

7) Any other misc comments/suggestions.


The key for me was going back over the answers i got wrong and the ones i thought were hard (i would
circle them when i while i was testing). Eventually you start to think like the testers think and can sniff out the
"trick" answers.

Also, the second time around, i barely studied (took 3 practices in the 3 months leading up to the test), but i
scored decently on them so i was very confident and relaxed going into the Oct test. The lack of stress just
made it much much easier.

[unknown LSAT tutor]


a) First, I got this advice from an LSAT tutor who has scored 180 4 times in a row, she is brilliant, and her
strategy is incredible. DONT TAKE PRACTICE TESTS WITH ONLY 4 SECTIONS. Better yet, she
recommended, take practice tests with more sections than the acutal LSAT, take them with 6 sections!!!!!!!
If you practice with 5 sections, you get semi fatigued around the 5th section. If you practice with 6, you will
train your brain for a hard six and you'll get fatigued around the 6th section, so you will NOT BE FATIGUED
DURING THE 5 SCORED SECTIONS OF THE TEST!!! This is the absolute best tip anyone had given me for
LSAT prep, IT WORKS, DO IT!!
b) So incase no one else has told you, the older tests are very different from the new ones. With this in mind,
what I did is take my tests with using the newer tests, but used the older tests (starting with the oldest #7) as my
unscored experimental sections. This way, every question your answering is a real lsat question, DONT USE
ANYTHING BUT OFFICIAL LSATS, the "similar" or "like official" tests, dont require you to use the exact
same train of thought and deduction process. Let me also state that I got a few "free b's" on my actual LSAT by
taking every official preptest because some questions are essentially repeated with a minor tweek or two.

SoberHobo
1) What score did you get?
179 - Oct. 08

2) What books did you use?


Kaplan Comprehensive LSAT and LSAT 180 (I didn't like these too much. I would rather figure out my own
methods without getting caught up in their terminology).
Every practice test available on LSDAS

3) What prep courses did you take (if any)? Full length, weekend?
None.

4) How long did you study for, and under what conditions? (during school, during the summer, etc)
I started about a year ago before the Oct 07 exam with the Kaplan books and a couple practice tests, but didn't
feel confident about my score so I canceled. About a month and a half before the Oct 08 test I began taking
a practice test every night after work. Sometimes I would take two tests a day on weekends to build up
stamina. After the first few tests, I would be sure to stay within the time limits. Also, I didn't watch any TV for
the entire 4 months prior to the test (never watch much anyway but I had just moved and was too poor to buy a
TV). Instead, I would read a lot, especially dense material, like Supreme Court cases and political
philosophy. This really helped get my brain in shape.

5) How many preptests did you do?


About 40

6) What would you change if you were to do it again?


Not much. I'm very happy with my score, but I was very nervous at the beginning of the test. Luckily the first
section was experimental. It seemed so hard that I almost gave up, but just kept going, hoping that it was the
experimental. I missed a couple easy questions that I shouldn't have. I feel like I rushed through each section. I
had at least five minutes to spare in each one but didn't bother to review too much. Could have saved that extra
point.

nsd230
Read A Rulebook for Arguments (short paperback book) and A Concise Introduction to Logic by Patrick Hurley
(an intro to logic textbook) then get to work on questions. Forming the base understanding of logic was the way
I went from a 159 on my first test to a 180. As stupid as it sounds, if you become a student of the LSAT you will
do well on it.

Roo
1) What score did you get?
178
2) What books did you use? (Kaplan, Powerscore LRB, Powerscore LGB, etc)

The Bibles (which I went through 2 times a piece), Superprep, the two most recent LSAT books containing 10
tests, all other released practice tests.

I did the logic games in LSAT180 but skipped the rest - its crap.

3) What prep courses did you take (if any)? Full length, weekend?

None

4) How long did you study for, and under what conditions? (during school, during the summer, etc)

Four and half months, while working and in graduate school.

5) How many preptests did you do?

25

6) What would you change if you were to do it again?

Try to relax the morning of the test. I was too stressed and I think it probably lost me a point. Also, I would have
started off with Superprep so I could get in the mind of LSAC to begin with. As it happened I used it towards
the end and it was not a lot of help.

7) Any other misc comments/suggestions.

1. Get obsessed. I researched everything I could about the LSAT, including reading journal articles about
the test itself and standardized testing in general. I got obsessed with every little detail ( i.e. How well do I
work in a too-cold room as opposed to a too-hot room? What about a noisy room versus a quiet room?). Cover
every detail, figure out how many cups of coffee put you at your prime, and what kind of pencil eraser works
best. Ridiculous, yes. This isn't a shuttle launch, after all. But treat it like one and you're less likely to have
unexpected little things throw you off on test day.

2. You need a cheerleader or two. Get someone important in your life - friend, sig other, parent, whatever, and
make sure they know how important this test is to you. This is someone to report your progress to on a regular
basis, who understands the difference between getting a 171 on a PT and a 175 and will help you celebrate.

3. Logic games were my Achilles heel and I knew it. When I started, I couldn’t finish one in under 12 minutes. I
put aside everything else and did logic games only for about a month. I made photocopies and did the
same games over and over. Focusing intensely on my weakness helped a lot.

4. On LR - cut out the questions you have trouble with and lightly pencil the right answers on the back.
Keep them all together and once you have a good stack go over them. Make sure you can explain out loud
why the right answers are the right answers, if you can't, you have to find someone who can explain it to you.
This board used to be a good resource for that but now that LSAT questions cannot be posted I'm not sure what
to do.

5. Reading comp. Forget the nonsense about reading the economist, etc. If you have reading comp issues then
by the time you reach test day you should have completed every single released LSAT reading
comprehension section under the sun, and should understand all of your mistakes on questions perfectly.
That's the best prep, IMHO.
6. If you're taking the LSAT in the morning, make sure you wake up early for a week or so to put your mind on
the right schedule. For seven days before the test I woke up at 6 am and took an LSAT around the same
time that I would be taking it on test day. Especially important for non-morning people.

7. To motivate myself I would imagine opening my email some day in July and seeing a 165. All of my
dreams of top schools would vanish at that point (thanks to a not so stellar UGPA). This provided me with
enough motivation to get out of bed early to study and do practice tests, etc. Especially if you are a self-studier,
you need to come up with some similar motivational imagery/mantra to keep you going.

Last thought on LR and RC - doing well on these sections involves really knowing the nuances of the
meanings of certain words and phrases. Focus on that fact in your prep. I found that certain words did not
really mean exactly what I thought they meant, and this made a big difference.

bgc
1) What score did you get?
179 - June 08 [bgc ended up going to YLS]

2) What books did you use?


Princeton Review, Powerscore Bibles, Master the LSAT (only RC), and GetPrepped's Ace the Logic Games
(Mostly for practice games).

Like most people, I highly recommend the bibles. The PR book was the first I read and was useful to get an
overview of the test but not groundbreaking. Ace the Logic Games has a large number of good games sections
on which to practice without spoiling real tests.

3) What prep courses did you take (if any)? Full length, weekend?
None

4) How long did you study for, and under what conditions? (during school, during the summer, etc)
Three Months. I work but didn't have many other obligations in that time.

5) How many preptests did you do?


All of them, in one form or another. I probably did 30 as actual timed tests and used others to provide the fifth
section or section practice. I did every games and RC section but left some LRs untouched.

6) What would you change if you were to do it again?


Apart from not making a stupid mistake on the LG and missing a 180, not much. I would start making notes
about my physical state and eating patterns sooner so as to have a larger sample size when trying to regulate
my body with sleep and food.

7) Any other misc comments/suggestions.


For me, volume of practice tests was key. My diagnostic was a timed 164 (June '07) and my second PT, after
going through the PR book, was an untimed 180. I didn't have major problems with the concepts so getting my
timing down and getting used to taking the test was very useful.

One thing I started doing towards the end was taking detailed notes about my sleeping and eating patterns, my
physical feeling during each test, and whatever other factors seemed significant. When on my game, I was
pretty consistent between 177 and 180. I have never been so aware of my own mental state as when studying for
the test and found that awareness to be valuable on test day. I was able to figure out, for instance, that taking a
day off before the test was NOT the right move for me. My practice results were better when I was on a roll.

Until one is solid on the concepts, taking a large volume of practice tests might not be useful. Once you
have it figured out, however, it really is all about practice. I was able to try a few different strategies for RC and
LG with enough time to get a sense of how they work.
[…]
RC was also my hardest section but I only missed one this time around. The thing that helped the most was
practice. It enabled me to experiment with different amounts and types of note taking and figure out what
would help the most without getting me in trouble with time.

180Splitter
1) What score did you get?

180.

2) What books did you use? (Kaplan, Powerscore LRB, Powerscore LGB, etc)

I started with Kaplan Complete (very incomplete, but simple and a decent introduction to the lsat) and the
Kaplan logic games book (an OK book). I then studied the Kaplan 180 book (the logical reasoning section is a
total waste but the games are good practice and the reading comp is OK). After that, I took ~25 practice tests
and reviewed every problem I missed at *great* length but my score was fluctuating wildly all over the 170's so
I buckled down and took Blueprint. Blueprint helped me by giving me methods to more quickly answer
questions I was already getting right so I had lots of time for hard questions. It also helped me focus and
kept me doing a lot of practice problems.

3) What prep courses did you take (if any)? Full length, weekend?

Blueprint full length.

4) How long did you study for, and under what conditions? (during school, during the summer, etc)

I studied during the summer on my own and then took blueprint full time, doing most of the homework.

5) How many preptests did you do?

Around 30 total, ~25 on my own and ~5 with Blueprint.

6) What would you change if you were to do it again?

I would have taken blueprint from the beginning and added a fake experimental section before every practice
test to replicate real conditions. I only did that occasionally. On some practice tests, I checked my answers after
every section (giving myself a small break). Not surprisingly, I did better than average on those! Practice tests
really ought to resemble the real thing in every way.

7) Any other misc comments/suggestions.

Preparing for the LSAT tests not just your intelligence, but your character. Do you have the integrity to never,
ever stop reviewing every question you missed until you understand why you missed it and will never miss a
similar question again? Will you be honest with yourself and take practice tests under realistic conditions, with
three sections in a row, followed by a short break and two sections? Do you take the time to bubble in the
answers as part of your 35 minutes per section? You aren't giving yourself extra time, right? Right!

The week before the test, I got up at the same time I would on test day and ate breakfast.

The day before the test, I drove the route to the testing center as if it were the day of the test. I went to a
room at the center and sat down in a chair, and thought about coming in the next day and getting a 180.

I don't drink caffeine and didn't before the test. I did pack a survival kit with many pencils, sharpeners,
Powerade, and Snickers marathon bars.

You too can do well on the LSAT. Take it seriously, as your best score will get you into a better school than your
mediocre score or get you scholarships where you were already planning on going.

GygesRing

1) What score did you get?


170; 180 (I retook)

2) What books did you use? 10 Actual, Official LSATs; 10 More Actual, Official LSATs.

3) What prep courses did you take (if any)? Full length, weekend?
None.

4) How long did you study for, and under what conditions? (during school, during the summer, etc)
Took a prep test every few days leading up to the test, would review the answers that I got wrong or had
marked as difficult while going through the test (didn't want to miss understanding a question type just
because I got lucky during a practice test.)

5) How many preptests did you do?


~17. (12/5)

7) Any other misc comments/suggestions.


Stay. Calm.

I was lucky enough to have a father who was able to explain any logic game to me quickly and helpfully, so I
didn't buy any review books, but the review books I did look at were much less helpful than an actual person.

Based on that, I'd have to recommend taking a class, or at least going with one of those options where you can
spend time at the test center and get questions you got wrong explained to you. I didn't take one (no time or
money), but I've heard the best things about Kaplan and not the Princeton reviews.

letylyf
1) 180

2) I used a Kaplan review book that was a year old and bought one of those 10 practice test books. I also spent
hours searching online for practice tests (even just individual questions) and explanations.

3) I didn't take any courses, so obviously it's possible to score perfect without them, but I do recommend them if
you have the leisure (time/money are always problems).

4) I took last year's December LSAT and probably studied at least an hour every day from mid-September
and solidly through November. I was lucky enough to have a job in a relatively quiet environment with
enough time on my hands to study, so that was a big factor in motivating me to study sufficiently.

5) The 4-5 that were in my book, about 8 in the ten actual tests book, and maybe 5 or so I found from random
sources (Kaplan and Princeton both sponsored practice tests at my university, looking online, asking friends
who've taken it for their old tests, etc).

6) Not a whole lot.

7) Actually I don't recommend taking time off from work or school solely for the LSAT. A lot of people get
burned out, and then you're almost worse than when you started.

I was lucky enough to find an amazing guy and we started dating the week before the LSAT, and I went from
extremely stressed and freaked out to relaxed and even semi-confident. I know there's not a magic formula for
everyone, but it helps an awful lot not to be stressed and constantly worrying taking the test itself. I actually had
time to check my work - twice - on every section because I was so focused. (I'd never even approached a 180 on
practice tests, funnily enough. A 177 was the best I'd previously done.)

I guess my advice here is don't push it. Study hard, discipline yourself, but create for yourself the best test
conditions you can... for me, that meant not taking any practice tests in the few days leading up to the real test.

[from another thread]

Reading fast helps enormously. My best advice on RC has always been to read the passage first - once, quickly,
and efficiently. If the class can actually teach you to read faster and comprehend what you're reading, it is
without a doubt worth it.

The more time you save on reading the passages, the more time you can afford to spend on the question and re-
reading a couple lines. Also, the RC sections seem to be getting denser and longer, so any investment in
improving this section is a wise one.

Depending on just how good you think the class is, I'd recommend it.

About the word/minute reading thing, I have no idea! I don't measure it like that. But if you can read (and
understand) a passage in a minute, you're set for the whole section.

lightIt
(not useful)
1) What score did you get? 180

2) What books did you use? (Kaplan, Powerscore LRB, Powerscore LGB, etc) Kaplan

3) What prep courses did you take (if any)? Full length, weekend?
Kaplan Night Course Weekly

4) How long did you study for, and under what conditions? (during school, during the summer, etc)
I studied while at school. I did a few problems a night about three nights a week for a few months

5) How many preptests did you do?


Maybe four or five

6) What would you change if you were to do it again?


I would Not take the course I would just get help on the logic games

7) Any other misc comments/suggestions.


I started at a 169 with absolutely no prep...I would say if you score at least 165 or thereabouts without studying
then a prep course is a waste of your time unless you are just too unmotivated to study on your own. The only
area it was remotely helpful was the logic games, and you could learn the Kaplan logic games strategy, or any
other Kaplan strategy, from a book just as well and much faster.

Lyrrad

Figured I should post here. Hopefully it's helpful.

1) What score did you get?


180 (PrepTest Average: 175, 176 in the few days running up to the test)

2) What books did you use?


Regular Kaplan Book (Premiere), Powerscore LR/LG/LGSetups, Kaplan 180

3) What prep courses did you take (if any)? Full length, weekend?
None

4) How long did you study for, and under what conditions? (during school, during the summer, etc)
1 Month. Nothing else to do that month except study.

5) How many preptests did you do? 27

6) What would you change if you were to do it again?


I wouldn't buy the LG Setups or Kaplan 180. I didn't use the LG Setups at all, and only looked at the tips in the
Kaplan 180 book.

7) Any other misc comments/suggestions.


a) Get the LSAT Proctor DVD. It's helpful.
b) Don't worry about not getting much sleep the night before. Just make sure you get rest the few days before.
c) Never be satisfied with an almost perfect score when Practising. Look carefully at all mistakes you make
on a PrepTest.
d) Don't over-study. I got burned out a couple weeks in.
e) Slow down studying the final week.
f) Understanding the Logic Games is more important than diagramming it the same way they do it in the
PowerScore book. If you know you can remember a key rule, you may not need to write it out.
g) If there's an either-or in a Logic Game, consider two diagrams, one for each case and its implications.
This was necessary to breeze through the Sept 07 LGs.

rucoach

1) What score did you get? 180

2) What books did you use? Both Powerscore Bibles and Princeton Review's LSAT Workout

3) Prepcourses? None

4) How long did you study for, under what conditions? 3 months, but not too much in the last month because of
work. 6-8 hours per week, a little less the month before the test. I try to study in quiet areas and always in timed
conditions.

5) How many preptests? 18

6) What would I change? I would travel in a time machine and beat my undergrad self for shackling me to a
2.94 GPA.

7) Comments? Go through the tests after you are done. Figure out why your right answers are right and
wrong answers are wrong. I would do every choice of every LG question to see why they were right or
wrong. Practice doing whole tests under timed conditions. Eat well on test morning, drink juice or water (no
coffee), and get in a rhythm on tests. I was taking a test every 4 days (spending the 3 days in between to fix
whatever section posed the most problem) by the end, so the real one just felt like part of that routine.

Haribo

Yay I've always wanted to post in this thread, and now I can!

1) What score did you get? 180

2) What books did you use?


LSAC practice exams (obviously recommended)
Kaplan 180 (not recommended)
Kaplan Games guide (I used this mostly for distracting myself from doing another set of boring LR/RC
questions, because I really like the games. It didn't help me much as I never had problems with games, but their
diagramming suggestions for sequencing games are excellent.)

3) What prep courses did you take (if any)? Full length, weekend? None.

4) How long did you study for, and under what conditions?
I took my first diagnostic in early September, and had a 175. At this point my biggest concern was my score not
dropping I studied on and off, probably between 2 and 8 hours a week, until 2 weeks before the test when I
got scared and did approximately 1 full practice test every 2 days. At this point my score still was pretty
inconsistent, and I could miss up to 4 in pretty much any section except games.

I ended up reading Voyager's guide to RC about 4 days before the test, and in passing his guide to the LR
section, and it helped me improve my consistency GREATLY.

I basically practiced on my bed, with a digital watch, under relatively sketchy timing conditions. I took a lot of
breaks and didn't do full practice tests often, at least not until the end (despite my best intentions.)

5) How many preptests did you do? 20-25 total

6) What would you change if you were to do it again?


Nothing! Not stress as much waiting for my score I guess...

7) Any other misc comments/suggestions?


I think I had a pretty easy time with the LSAT because of things I've been doing for a long time: lots of
reading, and when I do read I'm fast and can skim over details well if they aren't important, and an enjoyment
of logic puzzles, starting as a child and continuing today with those cheesy Penny Press games.

For RC, while I didn't write everything down on the RC passages, taking a few seconds to note the conclusion
and focus on the flow of the argument helped my timing. For LR, realizing that the right answer must
always be supported in the text, and really focusing in on the details of the argument and the different
parts of the stimulus made me go from missing between 1 and 4 per section to confidently answering them all
correctly literally overnight (the last 4 tests I took, including the real one, I finished with -0/0 on LR.) (I picked
up these tips from Voyager's guide which I highly recommend!)

In general, I really like tests like the LSAT and tried to have fun with it. It was very nice when everything
started clicking for me, and I went into the test with a lot of confidence because of it.

[from an email]
First of all, a caveat: I didn't use Penny Press puzzles to prepare, but rather see a number of similarities between
the LG section and some of the PP logic games puzzles. Also, I've always enjoyed the PP games and played
them quite a bit at various times in my life (for example, to keep me awake in boring lecture classes.) The
specific puzzles I would recommend are the actual logic games puzzles - very similar to the LG section. I'm
trying to find a sample but about to run out of time on the internet cafe... I'll look for it later. As far as chess
puzzles and mathematics, I don't think it would hurt, but is probably not the most time-effective way of working
on things. If you're genuinely interested in chess or math, go for it. Otherwise, I'd not bother.

[from a post]
The biggest difference I made to move from 177 to 180 was how I approached LR problems. I was like you,
missing one or two in each section, and there seemed to rhyme or reason to which type I was having trouble
with. It was just two hard questions (or sometimes, 1 hard question and 1 easy question that I misread and
turned into a hard question.) For these, I would be stuck deciding between two similarly correct answers, and
end up guessing.

My turning point was when I realized that I should never, ever guess on an LSAT question. They are designed
so that there are 4 wrong answers and 1 right answer - not 1 right answer and 4 almost right answers. If you
cannot come up with a reason for every answer choice you cross you, you don't understand the problem. Try
going back, looking it over, rethinking it, diagramming it, restating it, whatever will help you (for me, I found
rewriting it in simple English helped, removing extraneous words and helping me see the patterns more clearly.
I would also spend up to 5 minutes on a single LR question if it took me that long to determine the single
correct answer, and not give up when I didn't figure it out immediately. YMMV.)

Anyway switching my attitude was what got me those last 3 points. Promising myself that I wouldn't guess on
any more problems, realizing the mistake didn't lie in the test or the questions but with my understanding - it
sounds simple and stupid but I saw results immediately. It sounds like this is a problem you're having, so
hopefully attacking those LR questions a different way will help you.

Also, just to add my own experience to what other people are saying - I did zero 5 section practice tests before
the LSAT, and I also tended to split the sections up and work on them individually. I wasn't strict about timing,
for the most part, but I do admit to using the dull pencils trick - it's so much more satisfying to fill in the circle
that way!

Framboise

1) What score did you get?


180 (Sept 07)

2) What books did you use?


LSAC practice exams (the single most useful study aid out there)
Kaplan 180 (highly recommended)
Princeton Review (not at all recommended - read about 10 pages, realized it was a waste of time and never
looked at it again)

3) What prep courses did you take (if any)? Full length, weekend?
None

4) How long did you study for, and under what conditions?
I took a diagnostic test at the beginning of July and scored a 168. Then I didn't study for the next 2 months. I
began studying in the beginning of September. I spent about 4-6 hours a day studying Mon-Fri for three weeks.
(I go to a semester school, so classes hadn't started yet and I was done with my summer job.) On weekends, I
either rested or spent at most 2 hours studying. The week before the test, I took two 2 practice tests, but other
than that, tried not to think about the LSAT at all. Taking this break helped me significantly reduce my stress
level.

5) How many preptests did you do?


Between 10-12. I tried to take all of my preptests under conditions as close to those of the actual test as
possible. Always doing all the sections in one sitting, timing myself with a non-digital watch, etc.

6) What would you change if you were to do it again?


I would not spend 2 months of the summer worrying about not having started to study yet. Either start studying
early or accept the fact that you're prone to procrastination and won't start studying until the last possible
minute.

7) Any other misc comments/suggestions.


Make sure you know about all of the weird LSAT regulations, e.g. no cell phones, no digital watches, no pens,
no hoods, etc. etc. And put together your clear plastic bag two or three days before the test, so you're not
worrying about what you need to have with you the night before or the morning of.
If there are many test centers in your area, ask people who've already taken the test about the conditions at
the various centers. My test center had great conditions - lots of table room, comfortable chairs. But I've heard
horror stories from people who went to other centers in my area.

JDewey

Thought I would give my bit of advice. I took a more holistic than most people I think.

Here are my top three bits of advice.

1) Read all different kinds of information. I would read The Economist, The Smithsonian, and Scientific
American, daily. I think this is what helped the most, and reading should be a top priority. You can re-wire the
neurons in your brain to better handle written words if you read all the time. Most people's brains are probably
set in "T.V. Mode" and unfortunately there is no video section on the LSAT. Turn off the television, start reading
dense material, this will help you across the board not just on the reading comprehension section.

2) Pick up a good book on INFORMAL logic. I hear people saying that formal logic important, believe me,
for this test INFORMAL logic is actually what you want to study. I think people are confusing the two. I am
particularly fond of "Informal Logic: A handbook for critical argument" by Douglas N. Walton.

3) When you are studying, figure out why each answer choice was wrong, as well as why the correct one
was in fact correct. This is kind of common sense but the tediousness of doing this causes it to be often
ignored.

Cheers

Unstoppable

1. 180

2. No books, except for those from the LSAC containing just prep tests.

3. None.

4. I began studying in August and took the test in December. I did not work or go to school during this time, and
I had to move home with my parents to allow that. For some, that might not be possible, and for many more, it
would be undesireable. For the latter among you, I suggest you take the time to assess what is really important:
a test that can and will contribute significantly to determining the career opportunities you will have for the rest
of your life, or your personal lifestyle for the next few months? If you lean towards the latter, I suggest you re-
evaluate your commitment to law school to begin with...

My preparation consisted almost entirely of prep tests, mostly under very strict conditions. Because four
sections does not really prepare you for the six you will face on test day, I usually did them in pairs: four
sections (a full prep test), then a 15 minute break, then four sections (another full prep test). This is more
extreme than the actual test, so if you can master that level of endurance, then the actual thing will be a piece of
cake. I also used the same equipment I would be using on test day, down to the same pencils, sharpener, watch,
et cetera. This is important because you do not want anything to be different on test day so that it might throw
you off. Some people advise taking prep tests in a public area where there are other people around such as a
library. This may be a good idea if you are prone to being distracted by that sort of thing, but I just took my tests
at my desk at home and did not find the change in environment at all distracting on test day.

The other important thing, and this is echoed in the posts of a few other top-scorers, is you don't just take the
prep tests, but you have to review them thoroughly. I would say that I spent about twice as much time reviewing
each test as I did taking them. Any question I got wrong would get exhaustive attention in this review; I would
analyze exactly how I approached the question, why that approach failed, and what approach would have
allowed me to select the credited response. I would similarly pay great attention to each question that I had
marked as 'unsure' during the test. It is important to see how you got it right, and how you avoided getting it
wrong (perhaps narrowly) rather than just be content that you got the correct answer. That said, there may be
questions for which the answer does not seem as cut-and-dried, even after extensive consideration, as perhaps it
ought to be. Do not dwell extensively on these; whether the problem lies in you or in the test (neither is perfect,
I assure you), there should not be so many of these as to seriously affect your score on test day. In fact, I would
say I only encountered a question which I could not reconcile about once every four prep tests. If it is happening
a lot more frequently for you, then you may be missing some important rule or concept which is essential to
understanding those questions. Try to look at them as a group and see if they have common elements, and
perhaps discuss them on a forum like this one where other people may be able to illuminate the issue that is
causing you problems.

5. In the end, I did between 30 and 40 prep tests. If you are going to leave some out, then leave out the older
ones, since it tends to be the case that the more recent the test, the more relevant it is to the one you will have on
test day.

6. I would have investigated my test center a little more thoroughly. As it turned out, my center was fine (except
for the fact that they did not get started remotely on time,) but it was one area that could have caused trouble
and I essentially just "lucked out" in that regard. Other than that, as I am completely satisfied with the outcome,
I would not change anything.

7. I was worried about taking the test in the morning, especially with a 1 hour plus drive to the test center, so I
switched to a sleeping schedule whereby I would sleep from about 6 p.m. to 1 a.m. about two weeks before the
test. This allowed me to take the test in "my afternoon," and worked quite well as I never felt remotely tired or
unfocused during the test. However, I don't suggest doing this unless you know how your body will react. That
said, if you are really not a morning person it is something to consider, though I'd suggest that you start
experimenting with adjusting your circadian rythm months ahead of time to get a feel for what you can expect.

Shadyb

[from a post]
Endurance is really important - I took two practice tests on many days leading up to the exam so that I wouldn't
get that tired burned out feeling at the exam.

I didn't run out of materials though, so I'm not sure what to tell you about that.

When you say you ran out, do you mean just of PTs? Because reading and working through some prep books
might be helpful - you might find the strategy that *clicks* and will gain you some points. My first diagnostic
was a 160 until I worked through a prep book and learned the strategies. Then try using them on the PTs; and as
others have said, make sure you understand why every wrong answer was wrong. If it's a LG, do it over from
scratch to drive the lesson home.
Since you've been studying so much, you've probably already done all this stuff, sorry!

[from another post]


Re Endurance, I would recommend taking 2 4-section tests in a day several times before the real thing. One in
the morning, break for lunch, then another full one. When your afternoon score is as good as your morning
score, you have enough endurance. The real exam is more tiring because you care so much about doing well,
and you want to feel as fresh on the last section as on the first.

[from another post]


go through a different brand of prep book. Do all the exercises. You might find a different strategy that works
well for you. Also, you need to take a lot of timed tests, as time is the biggest challenge. Your timing will
improve if you do. Don't worry that your first full length test wasn't as good as you wanted it to be. You need to
build up your stamina and speed. Some days I took two full length tests - it was horrible, but helped with my
stamina. Good luck!

[from a PM]
I only studied for a month (because I decided late to apply to law school, 2008 cycle in Canada - I went there
for a year then came to HLS as a first year...it's a bit odd). I wasn't in school or doing anything, so I studied 6-7
hours a day. On my first diagnostic which I did without any studying and untimed, I got a 160. I then carefully
worked through the Master the LSAT book - I read it cover to cover, doing every exercise and every section,
timed as recommended. I found that the strategies really worked for me (especially for LG) and that learning the
tricks was amazingly helpful. I did pretty well on RC from the start, but it was the section I had the hardest time
improving on.

After that, I started doing timed practice tests. I went over them and studied every question I got wrong. After I
had worked through the book, my scores jumped to the 170s - I didn't get below a 170 after completing it. By
doing practice tests I gained exposure to the different types of games and LR questions, as I'm sure you have. I
already mentioned what I did to build endurance.

Honestly, I really don't have a magic bullet...I took a couple of days off before the exam because I was getting
so sick of it. I just did a few sections of RC, which I was having some trouble with (at that point I was hardly
ever getting any wrong in LG or LR because they are essentially formulaic). I will note that I never got a 180 on
a practice test - I scored consistently in the high 170s.

I don't get nervous for exams, so I just focused on test day and did my best. I felt like a had a bad section (must
have been the experimental one!) but put it out of my mind and focused on what I was doing. I had no trouble
with time because of doing so many timed practice exams, and didn't get a headache or get worn out.

I feel like that was a big lot of useless drivel, sorry!!! Feel free to ask more specific questions if you want, about
the LSAT or HLS.

A 180 vs. high 170s really comes down to luck, so good luck!!!

[another PM]
I think working through another strategy book will really help you. I found that the strategies in Master the
LSAT worked great for LG and LR - they were quick and very accurate.

I took 2 practice tests a day for a couple of weeks - most days but not every day.

[another PM]
When I was working through Master the LSAT, it was my first look at the test, so I usually got a few of the test
problems wrong. I tried to figure out why, and what type of question it was. Usually I could see my mistake
from the answer explanations, but sometimes I also went back and looked at the section addressing that type of
question and figured out where I had deviated from their strategy. I did read every answer in LR, but not
carefully. Whether you should or not probably depends on whether you have time. I just couldn't get my head
around not reading some of the answers, it felt like walking into a trap.

danny

I scored a 178 on the September 2009 LSAT and hopefully have a few helpful suggestions on how to score well.

The books I used were all the available past tests which I think are the best resources and probably all that most
people need as far as materials go.

[Redacted.]

I also recommend taking a practice test every week. I took a practice test every Saturday at 8:00 am which
prepared me for the real thing and took away a lot of the anxiety on the day of the actual test.

During the week, I studied by drilling particular question types and game types, as well as practicing whole
sections, but saved the practice tests for the weekend.

I hope this helps, and good luck on the LSAT.

Woozy

I will record what I did here in the hopes that it may help some, but I would like to preface this by saying my
study plan would have been insufficient for the majority of takers. I had a great advantage in that my initial
diagnostic (after looking at the LG section of a prep book for 1-2 hrs) was already >170.

1) 180

2) A friend had taken the Testmasters course and I had access to the complete set of their books, which contain
all LSAT questions from mid 90s to about 2005 or so, organized by question type.

3) None

4) I studied for 2-3 months, about 5-10 hours per week. I was also working full time.

5) 5-7 PTs, 2 of those were crappy fake tests. Use only real LSATs! The fake tests made by test prep companies
do more harm than good.

6) Take more PTs. I made a mistake in having too few PTs to work with. Also, I should have been taking 5
section PTs.

7) I have a few thoughts about test taking and test prep. Many are not new, some are:
A) Try to be in decent physical shape. Sound body, sound mind. Don't go crazy, but at a minimum do 30 min of
physical activity a few times per week.
B) Take PTs honestly. Time strictly, use bubble sheets, etc. If you are dishonest here you only hurt yourself.
C) Think efficiency and speed. Every second needs to be spent productively. Try to move as quickly as you can
from question to question and from answer sheet to test booklet. It sounds simple and obvious, but it is not so
easy in practice to identify every wasted second. A few examples of things I did:
i) Practice turning pages and beginning immediately. Attack the questions. Feel a sense of urgency during
practice and the test. In LG practice setting games up as quickly as possible; often you can just skim the
opening paragraph. You should be able to immediately identify what type of setup to use.
ii) Late in practice when you have internalized your sense of timing, stop looking at a watch. Every second
spent looking at a watch is a second not spent on the LSAT.
iii) Use dull pencils - they fill an oval quicker.
D) Practice is the time to develop your focus! When you go through problems, try to move with a sense of
urgency and do not let your mind wander. Whether you are doing practice tests or just random problems, if at
any time you find yourself thinking about anything other than the problem in front of you, give yourself a
mental slap and bring your focus back to the test. If you are constantly thinking about random crap during
practice you will be thinking about random crap during the test. If you learn to do this properly you will be less
stressed during the test because you will not be worried about failure, nerves, stress, etc. since you will not be
able to think about anything other than the problem in front of you. I was very stressed out/nervous during the
12 hours leading up to the test, but once the proctor said "begin" the LSAT was the only thing going through my
mind.
E) Be flexible in your thinking. I have been tutoring a friend and he gets many wrong because he fights the test,
often deciding he likes some answer more than another because of something in his head, not on the paper.

Hope this helps someone out there. Best of luck.

[from another post where he copied most of the above information:]


5) Make sure you don't drink too much the morning of the test and bring a snack for the break.

[later post]
The main point of the dull pencil thing is not the few seconds it might shave off in a section but to illustrate the
mindset you need to have that every fraction of every second is important, and that during your practice you
should try to figure out the mechanics of moving fast, both mentally and physically.

It sounds like you need to set aside some time to do some serious dry runs. There are potential mental fatigue
issues with 5 sections done back to back that you need to work out before the test. This is separate from
tired/sleepy sort of fatigue, so you can't assume that being awake is enough. If you haven't been doing 3 sections
back to back, 15 min break, then 2 more back to back you need to start now.

[later post]
I made absolutely no marks on RC or LR passages. I only cross out answers I think are wrong and circle ones I
think are correct - the little letter next to the answer, not the whole thing.

That's not to say you should do the same. I did end up doing plenty of rereading, so I'm open to the possibility
that some strategic markers could have helped. Do whatever works for you.

[later post]
For all the LSAT discussion around here, I see surprisingly little about mechanics. Stuff like keeping the
scantron as close as possible to the booklet, using a dull pencil, flipping pages quickly, not looking at stuff other
than the test, etc. never gets discussed. People are rightly concerned with understanding the test, but they should
still spend some time thinking about taking the test.
Sure, saving every single second will probably only net a couple of raw points, but why leave them on the
table?

[later post]
I started off with a terrible Mcgraw Hill LSAT book where I did a diagnostic and went through their logic
games section. It at least taught me the rudiments of game diagramming. After that I got the TM books and
immediately recognized that I should dump the stupid book I had and work only with these.

I would have taken more PTs but I didn't have many - I hadn't found this site yet, I didn't even know what was
out there. I tried to take one PT per week, but couldn't get enough tests. I managed to get my hands on one from
the LSAC website and a couple from my friend near the end, which was a big help. Most of my time in between
running out of tests and getting my last few was spent doing games.

Basically, I'd say I spent about 65% of my time doing LGs from the TM books, 10% doing LR from the TM
books, and 25% doing PTs. I think it did help to have so many questions organized by type. I did a mix of timed
and untimed for the LG and LR, I did only strictly timed PTs. After each PT I would review questions I got
wrong and questions I wasn't 100% sure of. Not to toot my own horn, but it was usually not that many so it
didn't take much time. I probably spent 10-20 minutes reviewing each PT near the end, maybe 30-45 in the
beginning. However, I'm lucky enough to learn very quickly from my mistakes.

I thought about your bubbling strategy but it felt unnatural to me. It is definitely worth exploring, but I think
many people dare not try it because they fear running out of time with no answers bubbled. If you are quick and
disciplined it may work well for you.

[later post]
Anecdotal evidence and polls on this forum indicate more people score below their PT averages than above.
While many chalk this up to test day nerves, I have a different view. I noticed during practice that my PT results
and those of a friend I PTed with were not normally distributed. Although I did not have too many data points, it
seemed that the distribution of our test scores were not symmetrical about the mean, rather they had short tales
on the higher end and longer tails on the lower end. The results of the poll above mesh well with this view. For
me this realization meant one thing: I had a higher than expected chance of getting a significantly lower score
than I wanted. Most of my work in the last month (after I formed this theory) was done with the goal of
shortening this long low tail. One conjecture guided me: the long low tail would not exist if the test were
untimed.

That's why I'm so big on identifying wasted seconds. It is not enough to get to the point where you usually have
enough time to answer the questions, you need to get to the point where you have excess time even in a worst
case scenario. If you are essentially taking the test untimed, there is a very low probability that you will score
significantly below your PT average.

I'm convinced that a motivated and highly gifted person could get fast enough that he would have a decent shot
at a perfect score with 1/2 the time per section, so there is no reason you can't do the same in 30 min, and then
use the extra 5 minutes to hopefully move from decent shot to even better shot.

PoorOrpheus

Diagnostic: 160
September 2009 LSAT: 177
I studied for exactly two months. The first two weeks I went through a few chapters in each PowerScore Bible,
but quickly realized my score would improve quicker if I just went through a whole lot of tests. I took nearly 30
in that span of time and went over my wrong answers and questions I got right but had doubts about. My PTs
were always timed. I felt burnout coming on, so I only took a section or two each day the week before the actual
LSAT. My main points of advice:

1) Set aside 1 or 2 hours every day to study. It doesn't have to be in one chunk but get those hours in. Some
people can "cram" for the LSAT, but I think slow and steady works very well.
2) Chill out the week before. If you see you've reached a peak and are scoring lower than normal, back off for a
few days. You're probably just tired of taking so many PTs.
3) Take your PTs timed...
4) ...and take many. You want to walk into your testing center feeling like you've done this a million times
before...because you have. It's just another test, then. I immediately recognized the experimental on my actual
LSAT because it did not have the rhythm I had become accustomed to for that section. That's how attuned you
should try to be.

Good luck!

samsonyte16

Diagnostic: Low 160's


June '09: 176

I began studying in September of '08, took the LSAT in February, was dissatisfied with a 167, began studying
again and retook in June. In total, I probably studied for about 7.5 months. I used: the Powerscore Logic Games
and Logical Reasoning Bibles, the Powerscore Logic Games supplement, copies of every modern LSAT up
through PT56, and the book put out by LSAC that has three LSATs with explanations for every problem (I
forget the name now).

In the beginning, my worst section by far was logic games. On my first test I missed twelve on games and
around three on each LR and one or two on RC. I spent the first two months of my studying doing nothing but
games. I did the harder old games over and over again and eventually got to the point where I could do easy
sequencing games in about four minutes. Those two months took my score from the low 160's to the high 160's.

At that point, I began doing full practice tests. I averaged about two tests per week. I took them in fairly strict
testing conditions and spent my days off going over the problems I missed and redoing old games. Around
January, my LR scores suddenly jumped from around -2 per section to -0/1 per section, and I started averaging
in the mid-170s. I'm not entirely sure what happened on the February test as it was undisclosed, but I suspect
that nerves (and a cold/flu) got to me and caused me to mess up a game setup. At any rate, I knew right away
that I had to do it again.

I stayed away from LSAT for the rest of February and most of March. When it came time to start studying
again, I'd already used most of the practice tests so I began redoing the tests and again doing huge sets of games
on the off days. The only change I made compared to my pre-February studying was that I began giving myself
34 minutes instead of 35 for each section. Two nights before the June test I took my last new practice test and
got a 179.

In my opinion, the most important skill needed for self-studying is the ability to ferret out your own
weaknesses. It can be really hard to pick out trends in your own performance and hammer out your weaknesses.
sayan
diag: 165
sept 09: 176
PT average: 175-176

LR: bought kaplan mastery, read powerscore LRB (usually skimming), did kaplan mastery LR questions ONLY
by type to master each individually. in actual PTs, I aimed for 10 q's in 10min and hoped for 15 in 15. If I did 15
in 15 I could usually finish in sub-30 minutes.

LG: went to a thread where T14 broke down every logic game by type (similar to powerscore LGB) and
photocopied each game 3 times. I stapled all game types together (3 stapled piles of each type) and started with
linear basic and moved to grouping/advanced linear. I did each type over a couple of days and after at least 24-
48hrs, did the same game types again. I horribly sucked at LG thinking I would never do good but this strategy
worked flawlessly. I dominated LGs after a while and it was my easiest section. I only photocopied up until PT
42 or so.

RC: the hardest by far to improve in. I just practiced it as part of my PTs.

overall: I did every single PT from 9 to 57. i gave myself 40 minutes for the first 10 PT. I then went down to 35
for the next 10-15 or so. I then went even further trying to finish in sub-32 (or even sub-30). That was possible
for LR and LG but seldom the case for RC which was stubbornly stuck at 33-35 throughout my practice. finally,
I took timed tests in noisy cafes and quiet libraries. I did at least 20 PTs in different environments. By PT 35 or
so, I began using past PT sections as a 5th section. Usually it was RC because that was the most exhausting
section for me. I had a 2nd RC section on the actual test so I think it was a good decision.

The above regimen was done over a 3-3.5 month period with about 1-2hr/day average.

oneforship
First Diag (cold): 163
Feb '09: 175

I used the Logic Games Bible and the Logic Reasoning Bible. Luckily, time was never an issue for me, so all I
really used the LGB for was to get a consistent method of approach, and I was able to whittle that down to a
-1/0 consistently, and then -0 consistently the week leading up to the test.

The LRB I had to go through once completely, and then back to sections that were giving me problems. Every
LR section that I took and scored, I cut out the questions that I missed from the booklet, erased any marks, and
kept them together, and write the test/section I pulled them from. When I had 20-25 of those, I would run
through them as a "section" of only questions that had previously given me trouble. Typically, a second glance
would yield the correct response, although when I did have a repeat miss, I tried to really dig down and see
where I was missing. I think this was key for me to bringing my scores down from -4/-5 to the -1/-2 range
consistently.
For RC, I bounced around a bunch of methods before finally just settling on reading and answering, no
markups. I didn't really focus on RC until 1.5 weeks out, although I wish I had because I think I could have
picked up 2 or 3 more points here if I had tried. It was definitely what held down most of my PTs, I would be
cruising in every section and then out of nowhere a -6 or -7 in RC would just really bring down my score. It
wasn't until I realized the only way to fix that was to just do every RC section I had until I was consistent in
pulling out the answers and anticipating the questions. It didn't really click for me until 2 or 3 days before the
test, and I wish I had had more time for these, so definitely don't neglect them.

That said, for self-studiers, knock out the games first, it is by far the easiest section to pick up points in, IMO,
and it really is the most learnable.

Alexandria
I spent 7 weeks studying (but very diligently) and ended up with a 177. I'm not really sure what my diagnostic
score was, bc the first couple tests I took weren't official ones (got like in the high 150s)... they were from a
study guide (Barron's or something) that I eventually started to think was not very good. The first official prep
test (free one off the website) I took I got about a 172, but that was a couple weeks into studying, and I'm sure I
wouldn't have gotten that score right away.

I definitely liked the Bibles, like most people, especially LGB. For LG, learning to diagram in a consistent way
was key to getting fast. I didn't exactly use the LGB's method... I made some adjustments where their method
was counterintuitive for me.

LR was never hard for me (the way LSAC words things just made sense to me right away, so I didn't need to
categorize questions the way the LRB teaches you, in order to know what they wanted). So the LRB didn't have
as much of an impact, but it did help me on the two types of questions that sometimes tripped me up... parallel
reasoning and formal logic questions.

For reading, the things that helped me most were the book Kaplan 180 and just taking as many prep tests as
possible. I just had to get used to reading the boring passages and figuring out what they meant by the different
types of questions (sort of the opposite of my experience with LR... here I really did need to decipher what they
wanted from me).

The one thing I would have done differently is to have sprung for tons of recent prep tests, even though they're
more expensive than the books of older ones. I got a lot of 180s on the old ones, but they were harder to come
by on the recent ones (and I even got a 174 on one of the recent ones, which, frankly after having taken a lot of
the old ones and done so well, really freaked me out). The difference was definitely that reading comprehension
is harder on the more recent ones, and since that was my weakest section, I could have used more practice on
the newer RC sections. (On my actual test, all the questions I missed were RC.)

kajosa
Cold diagnostic: 147
June '09: 175

I studied for three weeks, starting with Princeton Review's Cracking the LSAT. I decided I needed more work
on logic games, but I didn't have much time, so I stupidly avoided the LG Bible and chose a less costly LG
book from McGraw Hill (I think). Terrible idea--I would never recommend that book. I also took some PTs--
maybe 7 or 8 over the three weeks. I did about half of them untimed, just working on fully understanding each
question. And of course, I thoroughly reviewed every question I got wrong until I knew why.

Someone else: “147 to 175 in three weeks? Sick! You forgot to mention which steroids you used though. ”

Ha! What I forgot to mention was how lucky I got. Dinos made LG a big blur of guessing, and a 175 was
seriously shocking.

?
I scored a 180 on the LSAT, and it was due in large part to advice I found on this site. In this article I hope I can
give a little back to the site and help others to reach their goals on the test.

If you are looking to strengthen your law school application, know that huge gains on the LSAT are possible,
and that your LSAT score is probably the main determinant of which school you’ll go to and what kind of
financial aid you’ll get. Some people mistake the LSAT for a test of intelligence, and thus assume that their
score won’t budge through studying. Never mind that new research suggests that what we understand as
‘intelligence’ can be improved through study. Some TLS users have reported a score jump of 20 points. Did
they suddenly jump from average ‘intelligence’ to the top 2% of college graduates?

I took a diagnostic a while back and I was pleased to see a 168. I thought, “All right! I’ll study for a couple
weeks and take a couple preptests to raise that to 170+, and I’ll be set!” Fortunately, I fell short of my goal,
because I was forced to realize how naïve I’d been and also how much more I could improve if I worked at it. I
came back later after a solid three months of filling my head with nothing but flaws in reasoning, the status of
women in medieval English law, and Abdul, Becky, Charm, Don Juan, Eiji, and Francine’s seating
arrangements. The effort paid off.

In TLS’s LSAT preparation forum you will find a wide variety of opinions about the best study methods. Some
posters insist that LSAT courses from the major test prep companies are worthless, and others say that they
really do help. At the very least, I can tell you that they are not necessary to score at the highest level on the test,
because I never took one. In my personal opinion, they may be a bad idea depending on how high you hope to
score. My own improvements on the test came from finding what worked best for me, rather than following a
method devised to work for thousands of students.

Much of what you need to know can be distilled to this:


-Though exceptional, plenty of TLS users raised their score by as many as 20 points over their initial diagnostic.
-This generally involves at least three months of hardcore study.
-To do the same, you want to take at least 30 official PrepTests timed properly and with a bubble sheet.
-You must thoroughly review your mistakes.
-Help your brain switch to text processing mode by reading in your free time.
-Find methods of attack that work best for you.

Create a Study Plan

TLS poster pithypike wrote up a very detailed post outlining a plan to assault the LSAT with special attention
paid to the Logic Games (LG) section. You should consider following it because many TLS users swear by it.
At the very least, you should create a similar study plan and stick to it.

Look to do at least 30 official PrepTests over at least three months. Some people do more PrepTests over a
longer period of time, but I would say 30 tests and three months are the minimums. Each week I would do a full
length PrepTest on Mondays, Wednesdays, and Fridays. After about three weeks, I started doing four games
sections that I'd seen before on Tuesdays and Thursdays.

When deciding which PrepTests to use, the more recent they are, the better. At some point around June 2001,
LSAC stopped writing the test themselves and handed the job over to ACT, Inc. Many people believe that the
test has gone through subtle but significant changes as a result. Not everyone agrees on what the effects of these
changes are. Since most people go through the tests in chronological order, I think there is some bias toward
thinking the new tests are “easier,” when in fact the person preparing is becoming more skilled. Nevertheless, I
found the Logical Reasoning and Logic Games sections to be easier on the newer tests, which Reading
Comprehension probably became more difficult.

If you’re on a budget, the earlier books of 10 PrepTests will be very attractive, but you should also purchase a
healthy number of the later tests. You might think that they’re overpriced, but they may literally be worth their
weight in gold if you get into the school of your dreams or if you’re offered a full scholarship because of your
LSAT score. I strongly recommend using newer PrepTests close to gameday so that you are accustomed to
them.

It’s easy to lose time or points because of errors on the bubble sheet. Practice with one! It took a long time
before I found a bubbling strategy that worked best for me in each section. I settled on page by page in LR and
LG, and question by question in RC. You’ll need to find what works best for you. Bubbling takes time out of
your 35 minutes per section, so you should practice with a bubble sheet if only for that reason. If you get to the
point where you can finish sections with time to spare, quickly checking your bubbling is an excellent idea.

In the next three sections I’ll tell you a little about what worked for me. What worked for me may not
necessarily work for you, but this should give you an idea of the kinds of things you need to be thinking about
when you’re reviewing your finished PrepTests. My advice may also be old news for you, but this is what I felt
would have been very beneficial to know. If you haven’t done a PrepTest or two yet, you can stop reading here
and come back when you’ve seen the test.

Specific advice for Logic Games (LG)

If you’re like most people, LG will be the most foreign section type for you. The good news is that a decent
number of TLS users eventually manage achieve consistently perfect scores in this section. As with most
everything in life (and other LSAT sections), repetition is the key to improving in LG.

Most people will probably want some system to give their thinking structure. Games were certainly my
weakness and it took forever for me to get them down. I originally looked at one of the Kaplan books, but
eventually settled on the Powerscore Games Bible. It’s not important to follow a system religiously, even if it
calls itself the Bible.

Although, repetition will do a lot of the work for you if you let it, you also need to think of general strategies to
improve. The broad instructions in the Games Bible should work for everyone. Start by diagramming the game
and fixing its rules and variables in your mind for a couple minutes. Then attack the questions. How you do this
is, in the end, up to you. As you gain more experience in games, you’ll find the wisdom in your book’s system,
but also what you can throw out or adapt. For instance, for the sake of clarity, a system may suggest you
represent a rule like this:
MN
D/A A/D

I found this to be too complicated both to write and read. It was easier for me to write it this way:
MN
DA DA

Find what works best for you!

For a long time, I was spending too much time writing out useless information that wasn't necessary to answer
the questions. Make sure you read and understand the entire question before you begin working on it. Let’s look
at a “linear game” to illustrate this. Linear games often have you placing people in order from 1 to 7, or factory
visits in chronological order on a schedule. Here’s a simple one:

A, B, C, D, E, F, and G must be inspected once each, once per day on consecutive days of a week.
E will be inspected directly before F. [EF]
B will be inspected before C. B<C
A is inspected second. A2
(etc., this game would probably have more rules, but they’re irrelevant right now)

My setup would look like this:

A B<C D [EF] G

A
1234567
FFE
EB
C

Now, let’s tackle the second question:

2. If D is inspected fourth…

At this point I would mistakenly stop reading the question and begin writing out the implications and
possibilities. If D is inspected fourth, then the [EF] block can only be in one of two spots:
A B<C D [EF] G
A D C/G E F
A D E F C/G
1234567

Look at that! I made a beautiful diagram showing both possible positions of [EF]! (Stacking two possibilities is
the best way for me to understand them.) I even made inferences as far as C and G. Now I can definitely answer
the question. Let’s continue reading it:

…the sixth day must be occupied by:


(A) B or G
(B) B or C
(C) E or A
(D) E or F
(E) A or F

I easily find the answer is D. The only problem is I wasted time worrying about C and G when I didn’t need to
go that far to answer the question. Read the whole question and preferably also the answer choices before you
start working!
If you’re like me and most other people, “grouping games” will be challenging. In these games, you’re usually
sorting people into two groups. In general, in grouping games you’ll help yourself by doing a couple quick
hypotheticals before tackling the questions. I made a habit of writing out the letters (HJKLPM, etc) next to each
hypothetical and crossing them off as I placed them.

Avon, Bodie, Cedric, D’Angelo, Ellis, Frank, the Greek, and Ziggy are all heading to a baseball game. They
will take two cars, a Mercedes and a Nissan, subject to the following conditions:
The cars seat four people each.
D’Angelo refuses to ride with Avon. ( D -> A , A -> D )
If Cedric rides in the Mercedes, so does Ellis. ( CM -> EM , EN -> CN)
If the Greek rides in the Nissan, Ziggy rides in the Mercedes. ( GN -> ZM, ZN -> GM )

We can start diagramming, but the only rule that always appears is D -> A.

ABCD
EFGZ
MN
D/A A/D
____ ____
____ ____
____ ____

Should our setup phase end there? I don’t think so, because we have almost no useful information to use for the
questions. The rules are probably not fixed in our minds at this point either. Take a minute or two to explore the
effects of the other rules through a hypothetical or two.

If CM, then, EM. Remember to cross of the letters as you place them in your hypothetical.

ABCD
EFGZ
MN
D/A A/D
C ____
E ____
____ ____

Only one space remains in the Mercedes. Z and G can’t ride together in the Nissan, so one of them occupies the
remaining space in the Mercedes, and the other rides in the Nissan. Our random variables B and F have no
choice but to ride in the Nissan.

ABCD
EFGZ
MN
D/A A/D
C G/Z
EB
Z/G F
We’ve filled in every square! This information should prove very useful.
(If you want to write it like I would, it looks something like this:
MN
DA DA
C ZG
EB
ZG F
Diagramming this way only works if it’s clear in your mind that each spot can only be taken up by one letter. If
you find this style confusing, then don’t do it. It helped me, but you should find what works best for you.)

Grouping games are often light on rules and heavy on the implications of those rules. During your “setup” time
in a grouping game, it can be very beneficial to work out some of those implications. It will save time if a
question asks, for instance, “If Cedric rides in the Mercedes, where does Frank ride?” Also importantly, doing
this will help you keep the rules in your mind.

I have sloppy writing. Writing out your diagram sloppily will continue to hurt you throughout the entire game.
After much pain and confusion on the PrepTests, I eventually discovered that my main diagram needed to be
organized.

Review your mistakes and identify pitfalls to avoid. Write down those conclusions and review them prior to
your next tests.

Specific advice for Logical Reasoning (LR)

Improvement in LR kind of took care of itself just by virtue of the fact that I did 35 PrepTests and thus 70
unique LR sections. If you do a similar number of PrepTests while reviewing and learning from your mistakes,
you should make some nice gains.

My strategy for improvement on this section was to do the first 10 problems in 10 minutes. I struggled with that
for a little while, but it gradually became second nature. Then I moved on to doing the first 15 in 15 minutes. By
the end of my three month plan, I could do 20 in 20 and even 25 in 25 if I was lucky. You should definitely try
to save time on the earlier questions since they tend to be much easier than the later ones. The early questions
can still throw you a curveball, however, so be careful.

Don’t be afraid to skip around, but make sure you do it intelligently. I would draw a big box around the
questions where I wasn't confident in my answer. I drew the box with a light line if it was a small doubt, and a
dark line if it was a big doubt. I would circle questions that I didn't answer. In general I circled parallel
reasoning without even looking at them because they’re best to answer last, in my opinion.

When I’d finished the last question in the section, I would go back and answer the circled questions, then look
at the dark box questions until satisfied, then the light box ones. Doing this will make sure you don’t get bogged
down during the section. You need to avoid spending 3 minutes agonizing over a difficult question and then
losing your opportunity to answer the easier ones.

Wrong answer choices on the LR section very often contain language that is too broad or (less often) too
narrow. Very commonly, if I was trying to decide between two answer choices, I could remind myself to check
the scope and immediately eliminate one.

10. A greater proportion of high school students these days are lazy compared with earlier generations.
Industriousness is both necessary and sufficient for academic success, and lazy students are never industrious.

Which of the following can be properly concluded, assuming the statements above are true:
(A) Blah blah blah
(B) Yada yada yada
(C) A greater proportion of high school students these days will find it impossible to succeed academically.
(D) A smaller proportion of students are industrious these days than in earlier generations.
(E) More high school students these days will not succeed in life.

Since this is question number 10 it’s still relatively easy, so we were able to quickly narrow the choices down to
two. Yet, because it’s question 10, and we’re trying to finish the first ten in ten minutes, we’re moving quickly
and may miss a key word or two. At first glance, D may actually appear more correct than C because its
language isn’t as “strong” and it requires fewer logical steps, but I hope you noticed that D is talking about all
students. C correctly limits itself to high school students.

Normally, noticing this would be enough to eliminate D, but if you want to think further, realize that there might
be more industrious elementary and junior high students than in previous generations. If this was the case, it
might outweigh the increase in lazy high schoolers. The moral is, pay very close attention to language indicating
scope.

I do think it’s a good idea to “prephrase” the answers on LR questions. Prephrasing is guessing the answer
before you’ve even seen the answer choices. This can be a great way to save time, especially on the earlier,
easier questions. Still, even if you think of a perfect prephrased answer, you should be prepared to throw it
away. Here is an example:

16. The only reason Don’s wife would leave him is her learning his secret identity, but she will definitely leave
him if she learns it. Don values protecting his secret identity above all else, and he would never intentionally
reveal it to her. Therefore, his wife will not leave him.

(At this point, perhaps you’re thinking to yourself, “What if she discovers it on her own?” You’re ready to look
for that answer among the choices.)

This argument is vulnerable to the criticism that it assumes:


(A) Don will not tell his wife his secret
(B) Don is a good husband and provider
(C) His wife can find his secret on her own
(D) Don will not accidentally reveal his secret
(E) Don will not act in a suspicious manner

Excellent! Our prephrased answer was among the answer choices. Let’s select C and move on… This is the
danger of prephrasing. If we find an obvious prephrased answer, we might pay less attention to the question
stem, and also ignore the correct answer. C is actually the opposite of what we’d hope to find. The argument
assumes Don’s wife CANNOT find his secret on her own. LR questions will often try to fool you like this.

Prephrasing is useful because, if you’d prephrased D instead, you’d have saved time and the question would
have seemed extremely easy. Someone with D in mind might be able to look at C and quickly see that it’s
incorrect. Prephrasing is very powerful, but you still need to be cautious when doing it.

Review your mistakes and identify pitfalls to avoid. Write down those conclusions somewhere and review them
prior to your next test.

Specific advice for Reading Comprehension (RC)

RC will ask you the same types of things about the passages each time. You’ll need to know the main point, the
author’s attitude, statements the author would agree with, understanding a metaphor the author used, and so on.
These questions will come up again and again so experience will help you find what to look for in the passage
while you’re reading it.

What helped me was realizing that all the answers should match each other. I refer to this as the “puzzle theory”
of RC. If you feel confident in your "main point" answer, you can use it to answer other questions that give you
difficulty. It works the other way around too. Maybe the main point is tough but you are confident in some
others about "the purpose of the passage" or "author would agree with." Consider this example:

4. The author would be most likely to agree that Billy Mumphrey’s downfall was primarily caused by:
(A) love
(B) deception
(C) his support of the conservative party
(D) his abandonment by key political allies
(E) his unbridled enthusiasm

You very confidently select E. Then later, you encounter this question that you find much more difficult to
answer.

7. Which of the following most accurately restates the author’s conclusion:


(A) The main character’s politics were more important than his attitude
(B) The main character’s politics and attitude were equally important
(C) The main character’s politics were less important than his attitude
(D) The main character’s politics and attitude were equally unimportant
(E) The dirty game of world diplomacy and international intrigue is unwinnable regardless of politics or attitude

You know “the main character” refers to Billy Mumphrey, and that this phrasing is simply meant to confuse
you. You remember an earlier question where you had to decide between politics and attitude. You look back at
number 4 see that unbridled enthusiasm had definitely been the cause of Billy Mumphrey’s downfall, and this
helps you to be reasonably confident in selecting C.

I often lost points on RC in PrepTests because of boredom. You'll be ruined if you get bored. You'll also be
ruined if you try to speed up, because if you feel like you’re reading very quickly, you’re probably missing
important information. If I get interested in the passage and read to understand everything in it at my own pace,
I can do well on it. This may vary from person to person though.

The TLS LSAT forum is quick to recommend The Economist for reading comprehension skills. This likely
stems from the perception that The Economist is written using more difficult language than many other
magazines. I don’t think any one magazine or book will prepare you for the reading comprehension section. It’s
a good idea to read a variety of magazines like The Economist, Scientific American, Foreign Policy, The New
Yorker, and others that seem like graduate degree holders subscribe to.

The RC section can cover, in any one test, topics as disparate as a 1950s German poetry movement, the
evolution of birds in a small corner of the Amazon, women’s landholding rights in the Magna Carta, and the
origins of Law & Wine Tasting as an academic field. Chances are you’ll find at least one of those hard to get
through due to boredom, convoluted language, or unfamiliarity with the topic. Reading more in your spare time
can only help. This may also have some benefit for unfamiliar topics in the LR section.

Review your mistakes and identify pitfalls to avoid . Write down those conclusions somewhere and review them
prior to your next test.

More on that underlined advice

This may be the most important advice I have to offer. Doing a lot of PrepTests is an obvious method to prepare,
but many people fail to review them sufficiently. I was guilty of it as well. When I sat down and thought a little
more deeply about some of my mistakes in LG, for instance, I was able to identify some serious flaws in my
method of attack. Here are the notes I wrote to myself over the course of those three months:
-Never take ANY section for granted
-Always make sure the answer choice fits PERFECTLY
-Be interested in every passage, game, and question
-Don't skip around on games questions if you want a -0
Instead, make a full hypothetical if stumped
-Read everything slowly enough that you don't miss anything
-5 question passages and games aren't any easier
-Grouping games, track categories on diagram
Write out letters for each problem, cross off placed ones
-One answer is 100% correct, others are 100% wrong.
If you can't see them that way, you're overlooking something.
Reread a tough problem critically if there's time. Don't rely on your memory.
-Wrong answers (esp. assumption) are often too broad. Watch scope
-GUESS INTELLIGENTLY - Letter used least within the section
-Explaining a discrepancy between two groups should show how something affects the two groups differently
-PAY ATTENTION WHEN BUBBLING! DON'T RUSH IT! BE CAREFUL!

I looked at this before I did each of my PrepTests. I also printed it out and brought it with me for test day. You
might find it helpful. I definitely recommend starting with a blank page (or text file on your computer) and
writing your own instructions to yourself.

Less obvious ways to prepare

It’s common sense that you would study the LSAT itself in order to prepare, but you may be able to improve
your score before you’ve even seen the test, or when you’re trying to relax after a PrepTest.

Many people say that studying philosophy is good preparation for the LSAT. That’s probably true. A well
rounded college education will prepare you for the reading comprehension section, at the very least. I’ve studied
a bit of computer science, and I felt that was very relevant. Several of my classes were devoted to manipulating
logical expressions and games, and I have no doubt that this helped me.

The classes I felt were most helpful were Discrete Mathematics (read: formal logic), Computer Architecture,
Theory of Computation, and Logic Design. This list is by no means exhaustive. All of these were in the
engineering department. Even within it, some were notorious for their difficulty. As such, I wouldn’t
recommend taking these classes solely to improve your LSAT ability, because they may well drag your GPA
down. On the other hand, if they’re already required or optional for your major, don’t blow them off if you’re
entertaining the thought of law school.

In terms of your attitude, you need balance when approaching the LSAT. You won't do well if you're intimidated
by the test, a section type, or question type. At the same time, you won't do well if you start to get cocky and
think that the test, a section type, or question type are easy. Each and every question needs to be treated as a
bomb squad member would treat a live explosive. Whether it's a hand grenade (LR question #1) or a nuclear
bomb (LR question #19 parallel reasoning), they can both blow up and kill you if you're not careful. I found that
I would miss questions if I fell into either extreme. Through practice you should be able to get yourself into the
correct mindset.

Some people recommend studying in a place with other people such as a library. It’s definitely wise to practice
somewhere with some background noise. It was inconvenient for me to go to the library, but I did practice with
the window open along a noisy street. One aspect of the test you can’t control is the noise and distractions in the
room, so you might as well be prepared for that.
Most administrations of the test will begin early in the morning. If you’re a night owl like me, then you should
train yourself to be up and functioning at this time of day prior to the test. Start waking up early and do a
PrepTest properly timed.

A psychology PhD may disagree, but it’s my understanding that it can be beneficial to study before going to
sleep. If you can study before bed, I read (in Scientific American!) that your brain will reprocess and digest the
information while you’re asleep. I didn’t do this, but I tried to take advantage of it the night before gameday by
doing problems from each section before hitting the hay. If you start having nightmares about logic games then
I would advise you to study earlier in the day.

Some people also find doing a warm up to be helpful. On test day I woke up bright and early and did about 8.5
minutes of fresh, never-before-seen problems from each section type. That meant 1 game, 1 reading passage,
and every fourth LR question in a section. You can do this pretty quickly and it should help your brain switch
into LSAT mode.

It goes without saying, but you need to take care of your physical well-being. Get plenty of sleep. I started
going to the gym while I was studying and I honestly think it helped. Exercise is supposed to be good for your
brain. If you normally eat nothing but junk food and drink soda by the case, consider investing in some healthier
fuel for yourself.

Retaking

LSAC keeps track of retakers and reports that most people only improve by a few points at best. They also show
that the higher your original score, the more likely you become to go down upon a retake. This means you need
to be cautious when approaching a retake. If you prepare for 3 months as best you can, take the test, and score at
your practice average, you should probably call it quits. Yet, if you score above 175 very consistently, and then
are shocked to find a 174 on your results page, then it may actually be wise to retake even with such a high
score.

If you don’t improve upon a retake, then you’ve shot yourself in the foot. On the other hand, if you can improve
by even one point, then you’ve helped your application considerably. Admissions personnel expect you to
improve simply through familiarity with the test, but since your highest score is the one that becomes part of the
school’s statistics, the candidate with a 165 and a 166 will probably fare better than the candidate with a 165, all
else being equal. I recommend reading TLS’s interviews with admissions deans for more perspective on how
they view retaking.

It’s incredibly important that you use your months of practice to establish what your real ability is on the test. If
you’re paying attention, you should learn your strengths, weaknesses, and strategies to score higher. You will
also learn what score you can expect when you take the real thing. Using this knowledge, you can evaluate why
you didn’t score as well as you expected. Was the test center noisy? Did you cut yourself some slack on practice
tests without necessarily realizing it? (For instance, some people take long breaks between sections, don’t use a
bubble sheet, fudge the section timing, or never practice with an experimental section.) Perhaps most
importantly, did you put in as much practice as you should have?

Depending on the answers to questions like these, you can decide if and when you should retake and what you
can do to ensure you do better next time. “I was unlucky” is not the right answer, because it excuses you from
taking any corrective action.

Cancelling

Some proctors and testing centers can be truly atrocious. Your proctor may call time five minutes early, or chat
on the phone with a microphone left on, or some other nonsense. You might also have some unimaginable
personal problem. Having been kept awake with anticipation, maybe you slept through your alarm and showed
up late. One TLS user claimed to have drunk two gallons of water the night before the test and severely
disrupted his bowels. In these cases, the option to cancel your score may be attractive.

The decision whether to cancel or not cancel is an exercise of your judgment, and admissions staff are well
aware of this. Thanks in part to US News, they have a strong incentive to look only at your highest score. Still, I
think it’s only natural to view Candidate A with a 155 and a 170 in a different light from Candidate B with a
cancel and a 170. When someone reads Candidate A’s file, they’d have to wonder why he didn’t cancel that first
score.

The other point to consider is that Candidate A with 155, 170 likely looks better than Candidate C with 155,
cancel, 170. Some schools (the top 3, for instance) state outright that they will look at your full testing record.
It’s your job to make sure it reflects well on you.

If you’ve prepared properly, I believe you’ll be able to make the right decision. When I took the LSAT for the
second time, an unbelievably loud protest was going on right outside the building, and it definitely affected my
concentration. I felt I’d performed well and didn’t consider cancelling, though I would have had a legitimate
reason to.

What they don’t tell you

Here you’ll find little bits and pieces of information that may calm your nerves or give you slightly more
knowledge about the test.

I hope that you are familiar with the concept of the experimental section. You may know that it’s impossible to
distinguish which section is experimental while you’re taking it. Test prep companies report, however, that the
experimental section has always appeared in the first half of the test. This information is not likely to help you
much, but it may demystify the test a little. On my second LSAT, RC came first, and in my morning mental fog
I felt like I may have gotten a couple questions wrong. I was very relieved when RC came again in the second
half of the test.

The LSAT’s last section is a writing sample. This section is unscored, but you should still try your best on it.
Even if it counts for very little (or nothing) in your application package, admissions staff do have access to it
and reserve the right to waitlist or reject you if you don’t take it seriously. Since it’s at the end of the test,
spending your remaining energy here cannot adversely affect your score, so give it an honest go. That being
said, very few people practice for it, and I certainly didn’t.

The TLS LSAT forum often gets questions about the “certifying statement” at the very beginning of the test. In
it, you’re supposed to promise not to disclose the contents of the test and whatnot. The proctor says something
like “Write, DO NOT PRINT, the statement.” My normal handwriting is sloppy printing, and if my life
depended on writing in cursive I’d have long since perished. On both of my LSATs I optimistically started the
statement in cursive and switched to sloppy printing midway through as my patience ran out. I didn’t run into
any trouble. LSAC doesn’t seem to care so long as you use your normal writing, but your proctor might. If your
proctor is a dictator then make sure to follow his or her instructions.

That’s it for now. If you still thirst for knowledge, head over to the TLS LSAT preparation forum.

bblobber
On the hardest questions, there absolutely will not be an answer that jumps out at you as the right one. There
should be 2-3 choices that you can eliminate on the first reading. Then you are left with 2 or 3 more that sound
somewhat reasonable. Your job is to eliminate the wrong ones by finding their flaws. You may be left with an
answer that doesn't sound all that great, but if you've done a good job of elimination, it will be the right answer.
This is what got me a 177 in December (-6 total; -1 on RC which had a ton of crappy answer choices).

kajosa
Did you sleep? That was what screwed me up the first time; I went from a 167 in September to a 178 in
December. Otherwise, I recommend just trying to stay calm. Once you're PTing that high, the key is just to keep
your nerves under control and make sure you get sleep. Clearly, you have the skills to do amazingly well, so
you don't need a lot more studying.

MURPH
I disagree with the study less, not more advice. Do every test you can get your hands on and go over each
answer choice with a fine tooth comb.
Better yet, put an ad in craigslist and offer to tutor students for a reasonable price. The actual pay isn't that
important but the experience of teaching and of having to take the time to really understand it before a tutoring
session or class is what give you the most improvement. Getting paid for learning the LSAT is better than
paying someone to teach you the LSAT. Some testing companies will hire you with a 170. I went from 170 -
175 dong this. PM me

tinman
I have a friend who did exactly what you plan. She got a 170 last June after studying a lot and scoring
consistantly in the high 170s. She then retook this fall and scored a 178. I don't think she studied much at all
between the two tests. I think she took a couple full-length tests during the two weeks before the fall exam. She
scored in the high 170s on those (after the break from June). I think for her the confidence from those last few
PTs as well as the restfulness that comes with not cramming before the latter exam were huge factors.

She was someone who does not always perform to her best under pressure. If you are like her, perhaps making
sure you are rested and relaxed before the next LSAT administration is the best thing you can do.

vampy
I PT'd ~177-178 and then I got a 174 on the real thing (first PT ever was 175). I think it basically comes down
to luck, and more than anything, stress. I was so stressed I could not sleep for 3 days before the test. I
recommend meditation.

[from a PM]
I think I am a case study of what not to do. I just thought and thought about the test and ended up not sleeping
for 3 days prior (I basically never scored below 175 in practice). For the upcoming test I've barely opened a
book and have been more occupied by other things. If I were to give you advice, it would be to get great sleep
the week before (no alarm clock) and do everything but think about the LSAT, at all. I tend to go over what if
scenarios: what if I get a 180! AVOID THAT. Anyway, no matter what you do you will probably sleep poorly
the final night, but if you had enough rest previously you should be okay. Also, stress releases a hormone which
helps you focus in the short run (<24 hours) but really beats you up over a few days. Thats why its so critical to
try to forget you are taking the test in the week beforehand.

You can also try meditation.

dovetail
1) What score did you get?

179 (Dec. 09)

2) What books did you use? (Kaplan, Powerscore LRB, Powerscore LGB, etc)

Powerscore Logic Games Bible

3) What prep courses did you take (if any)? Full length, weekend?

None.

4) How long did you study for, and under what conditions? (during school, during the summer, etc)

3 months, averaging 5 hours/week, while going to school (16.5 credits), working 20 hrs/week part-time

5) How many preptests did you do?

15

6) What would you change if you were to do it again?

I might have looked at the LR Bible. On test day, I got -3 on LR, and -1 on RC.

7) Any other misc comments/suggestions.

I played a lot of sudoku in the months leading up to my actual LSAT study. I get the same feeling playing
sudoku as I do when I'm working on the LSAT. I think the skills you have to learn to beat the harder puzzles are
pretty similar to the thinking it takes on the LSAT, and it's a fun way to upgrade your brain.

Also, lot of people think it's crazy, but I never eat breakfast on the morning of a big test. I always try to eat
healthily and consistently leading up to the big day, and then the night before the test I eat a big dinner with lots
of calories (to tide me over until lunch the next day [also, to help me sleep soundly through the night]), and then
go in to the test hungry. It's my theory that the body kicks the brain into overdrive when it's in a state of duress.
Think of it this way... if we were all hunter-gatherers, the best time to be thinking clearly would be when you
have to acquire food somehow.
It might be scientifically untenable, but it works for me.

[from a PM]
I would be happy to share more!

1. What time would you eat dinner? I ate at 8pm, the night before the test
2. Exactly what was the dinner (e.g. spaghetti)? Approximately how many calories was it? I went out for pizza,
and I must have had three pretty large pieces. I would guess they were 400-500 calories each, so it would have
been 1200-1500 calories.
3. What time did you go to bed? 10:30pm (where I usually was getting to bed at 12:30 or so).
4. What time did you wake up? 7:00 AM.
5. What time did you start taking the test? The testing center finally got things going at like 9:45. The hunger
really kicked in at the break, because everyone around me was eating. So I took one bite of a bagel, just to stave
off the hunger pangs. (I was afraid that if I ate more than that, my body would divert resources to my
stomach/digestion. Come to think of it, I'm not sure it didn't--since I missed 3 of my four after the break. But if I
hadn't eaten, the pangs might have been more distracting. Who knows!)

I wouldn't recommend pizza though... too much grease on the body. I actually had to rush through the first
section so I could get to a bathroom. Diarrhea. Luckily, it was an easy RC section. Also, it was the experimental.

1. What was your first PT score? If it was high, why do you think it was high? 158. And although that's not very
high, I only missed 2-4 questions on the RC and LR sections. Most of my mistakes (and subsequent
improvement) came from the games.
2. What did you do when you got a question wrong on a preptest? How did you make sure you wouldn't make
that mistake again?

For games: I would rework the whole problem (redrawing diagrams and everything), and then since there was
no time limit, I could always find out exactly why I had gotten it wrong. Usually it was because I had either
forgotten or misread a rule.

For RC and LR: after looking at the right answer, 9 times out of 10 I was surprised that I *hadn't* chosen that
one, since in hindsight it always looked so obviously correct. Probably 70% of my wrong answers came from
thinking I had a good answer, only to find that "D" or "E" was correct, and I just hadn't read and/or properly
considered it. It seems so dumb not to read every answer, but I found myself doing that relatively often. The
other 30 just came from my incorrect and unfounded assumptions. Training myself not to assume anything
unwarranted, while assuming everything that *is* warranted is the main thing I worked on, outside of avoiding
the dumb mistakes from the first 70% I talked about.

3. Did you get the feeling that sudoku helped you with all sections of the LSAT, or just the LG? All of them. If
you are playing a really hard sudoku, making sure not to write down anything (!), sometimes it forces you to
hold like 7 postulated things in your head at once before you can even consider finding a correct answer. That
skill helps just as much for games as it does for "sometimes X is Y, and sometimes Y is Z" type questions.

4. Did you do any outside reading that kept your brain in shape for LR/RC? Nothing for that express purpose...
but I was reading a lot for school throughout the semester. I'm a Japanese major, so we read a lot of things that
take a lot of focus to get through. Now that I think of it, Tale of Genji was probably a big help. It's over a
thousand pages, and it can get very convoluted and very boring sometimes. And there's the whole thing about
how the author never expressly names the characters--she mostly uses their public titles. And the real tricky part
of that is that people are constantly getting promoted, so their names change up to 7-8 times each. On top of
that, sometimes she doesn't even tell you who is saying what. I think the way the book forces you to read with a
really open mind (ready for your perspective to change at the slightest change of a detail, or when you finally
figure out who said what way down the line), as well as the way the plot changes so dramatically on small
easily-overlooked details trains your brain pretty well for LSAT type stuff. That was a convoluted sentence.

Zen of 180
When I first took the LSAT, I had been scoring in the 175 range on practice tests. Therefore, I was unhappy with
the 172 I received on the September 2007 test; on the day of the test, I felt it had gone well. Even though I was
in the LSAT 99th percentile, I had "bombed" on the last section, reading comprehension--now I know that
reading comprehension sections are the most susceptible to LSAT fatigue because they require you to hold so
much information in your working memory at once. Although I had taken many full official PrepTests, I became
mentally fatigued on the test day and couldn’t perform to the best of my ability.

The realization that test fatigue was my main problem is why deciding to retake the LSAT was correct for me: I
had pinpointed my problem and knew exactly what I needed to do so that there was a significant change in my
practice. Instead of accepting my 172, I went about securing the score I knew I was capable of. Over the next
six weeks, I developed the system of training for the LSAT that is the basis of Zen of 180.

To address my fatigue, I practiced longer than the actual LSAT and required the same level of accuracy from
myself in order to be satisfied. I followed a strict regimen of taking eight full sections--two official PrepTests in
a sitting--twice a week. After analyzing and correcting the errors (which would later become the basis of the
Zen task strands)I made in the logical reasoning and reading comprehension sections, I improved my average
practice score from 175 to 179.

It's worth noting that my practice not only addressed my issue of fatigue--the original problem that kept me
from scoring at my original practice average--it also improved my accuracy in ways that I hadn't felt necessary,
obviously a welcome side effect.

On the second test day, in December 2007, I was fully prepared to handle each section and, more important, the
entirety of the test. At the break, I bounced around the hallway while everyone else looked as though they were
on their deathbeds. Their perception of their practice, the test itself, and its importance made the LSAT an
imposing, exhausting, and terrifying ordeal. I, however, played games (and not just in the analytical reasoning
section) with the test because I understood its rules.

I attribute my perfect LSAT score of 180 to my hours of practice and the mentality I developed through training,
not to any innate ability. In my experience as an LSAT tutor with clients starting out from both the 130s and the
170s, I have seen that everyone is capable of improving their score through regimented practice which is
designed to address their specific weaknesses.

It is worth noting that in addition to the two problems I fixed while developing the Zen of 180 system, LSAT
fatigue and task-specific accuracy, there is an additional problem of reading speed and comprehension. If you
have the time to develop this baseline skill, start so now; hopefully if you get your Zen on with our training
schedule and target your weaknesses, you won't have to worry about taking the LSAT a second time.

[from another post]


"I just don't read fast enough, I can't finish the sections on time."

We get LSAT timing issues with a lot of our Zen students, so we compiled this list:
The Top 10 Ways to Increase LSAT Reading Comprehension and Speed
(before ever taking the LSAT)

1) READ DAILY, and be selective in what you read. We suggest the New York Times, as it is high quality
writing in a brief format and free to read online and print. We read the paper on the iPhone at least 40 minutes
every day on the subway, but you should focus on certain article types for LSAT purposes.

2) RE-READ any sentence or paragraph if you can't explicitly answer the question, "Why did the author put that
here?" This metacognitive process is one of the best ways to make sure you are still on the same track line the
author laid down for your train of thought.

3) USE op-ed contributions as proxies for analytical reasoning stimuli because they feature explicit arguments
and are shorter, like the LSAT prompts.

4) ASK yourself questions while you read the op-ed pieces:

"What part of the argument is this sentence? Conclusion, bridge, detail, context, tone, or irrelevant?"
"Are all the numbers cited accurately? How do the raw numbers and ratios interact?"
"How is the author trying to convince me?"
"Are there any gaps in the argument that make me doubt it?"

4) USE science, business, arts, and technology articles as proxies for reading comprehension, as they often
feature opinions from disagreeing experts in the same fields as the passages in the LSAT.

6) ASK yourself the same questions as in the op-ed pieces, but include some new questions at the end of each
paragraph or section:

"Whose opinion did I read and what did they say?"


"How did the author's tone differ from their sources?"
"How do the sources disagree with or compliment each other?"

5) PRE-PHRASE your own title and see how close it is to the actual title and subtitles (so don't look too hard at
the hyperlinks until after you've read it!). The Times has solid, succinct, and interesting titles, which also means
this strategy is great for checking whether or not you nailed the main idea of the article.

8) CHECK the answers to all of the above questions and mark the evidence in the text. This will not only make
sure you are correct, but also hone your skill to reference back to specific parts of the text.

9) TELL someone about the articles and opinions you read, as it forces you to verbalize the most important and
compelling pieces. If they stay interested, you're doing a good job.

10) TRANSFER these skills and questioning techniques to your LSAT practice and start using all of them while
taking the test. Actively reading--not just processing but concurrently analyzing--LSAT prompts during the first
read-through will reduce confusion, eliminate the need to re-read, and allow you to quickly eliminate incorrect
choices.

Each of which translates into more time for getting your Zen on at the end of the 35 minute sections!
s0ph1e2007
I think its probable I didnt get any wrong on RC in february and this is what I did.
I started out RC by reading through the reading comprehension bible and practicing with their massive
underlining, key term blocking, blurbing per paragraphing- method. Then once I really understood fully why it
was that i was underlining and boxing those things I quit making notes while I was reading. I just read slowly
and intensely and boxed maybe two or three terms.
I went immediately from -3 average to a -0 average on RC once I did this.

KibblesAndVick

1) What score did you get?


179 on the Feb 2/20 makeup exam

2) What books did you use? (Kaplan, Powerscore LRB, Powerscore LGB, etc)
LGB and lots of old practice tests. Old prep tests are the key material for mastering the LSAT imo.

3) What prep courses did you take (if any)? Full length, weekend?
I didn't take a prep course. They are way overpriced.

4) How long did you study for, and under what conditions? (during school, during the summer, etc)
I spent about a month and a half studying nonstop. Usually upwards of 5 or 6 hours a day. I would take a PT in
the morning, eat lunch, come back and review mistakes.

5) How many preptests did you do?


About 20

6) What would you change if you were to do it again?


It worked out well for me so I don't think I would change much.

7) Any other misc comments/suggestions.


I think that successful LSAT prep can be broken down into three stages. First, you have to master the material.
The LG Bible is absolutely wonderful for this. I didn't use the LR Bible so I can't attest to it. In the beginning of
your prep you just have to make sure you understand how the test works, what to expect, why the right answers
are right, and why the wrong answers are wrong. Second, you have to make sure you can preform under
simulated conditions. Once you have a handle on the material, I think you should take every practice test with
strict time limits and added experimental sections. Taken individually, LSAT questions are usually not very
difficult. The real challenge (imo) is being able to piece together 5 near perfect sections in a row without
running out of time. Third, and finally, I think that success on the LSAT comes down to showing up on game
day. There are a lot of people who can average in the 170's on their PT. There are also a lot of people who have
hit 180 on a practice test. The final step is being able to replicate that result when the game is on the line. This
means you have to get a good nights sleep before the test and find a way of dealing with your nerves. This
is something many people have trouble with. Personally, my strategy was to talk myself out of the
importance of the test. I would tell my friends I didn't care about the LSAT and didn't really want to be a
lawyer anyway. This was total BS, but it worked. I didn't freak out when my test date got postponed, I was able
to get 9 hours of sleep before the test, and I felt calm and collected while taking it. If you're capable of
hitting 170+ on practice tests, this is what I would stress most to you. You have to be able to remain calm under
pressure. You could be the most accurate sniper in the world and it wouldn't do you a damn bit of good if your
hands started trembling in the heat of battle.

tomwatts

Well, I teach the LSAT for the Princeton Review, so I'm a little bit of a weird case. I've been doing LSAT
problems (and explaining them to people) more or less continuously for about four years. But basically, in
addition to doing the same LSAT questions over and over and over again ad nauseam, I took PTs 57 and
58 to see the recent trends. I'd already done everything through 56, so all of that was familiar, and then I did
57 and 58 and would've done 59 had I not 180-ed on 58 (figured that'd be a good note to go into the test on).

But I really think that people underestimate the utility of doing questions repeatedly. It helped me.

[another post]
It is true that no matter what you do, the bulk of your time should be spent working and reviewing real
LSAT questions. Whether you self-study or prep with a class or tutor or whatever, you've got to do a lot of
work on your own, too.

[another post]
As I said on test day, this felt a lot like PTs 57-59, especially in the games. The preponderance of In/Out games,
the "switch a rule for an equivalent rule" question, the types of games and deductions and so forth were all very
reminiscent of the other tests in the past year. The rest of the test felt the same way. Now, PT 57 was hard. That
was a jolt, compared to previous tests; I suspect that it was the beginning of the new trend in game types and
such. Anyone who didn't work PTs 57-59 carefully and go over them carefully would've felt that jolt in
February, and that's probably a big part of the reason that people have been saying that February seemed awful.
If you only prepped off of PTs 1-56, you might have found February to be awful. But people who say that
February was much harder than 57 or 58 are nuts.

I do get, in hindsight, how presenting the juicers, mixers, and the rest could have thrown people off. I had a
huge advantage because I've taught the similar (and worse) game in PT 38 so many times that this game didn't
surprise me in the slightest. Anyone who hadn't done that game might've been at a small disadvantage. Anyone
who ran out of time on the last reading passage (which was the easiest) probably screwed himself over. And
there were easy ways to go wrong in the LR, too. However, I don't think that makes this test any different than
any other.

My conclusion is this: This test was another one much like the three that preceded it. It was hard, but the LSAT
is always hard. It wasn't unusually hard, nor was it exceptional in any other way. This is what the LSAT
normally looks like, with recent trends manifested well.

sharpnsmooth

1) What score did you get? 178


2) What books did you use? (Kaplan, Powerscore LRB, Powerscore LGB, etc) Kaplan & Powerscore

3) What prep courses did you take (if any)? Full length, weekend? None

4) How long did you study for, and under what conditions? (during school, during the summer, etc) 6 months, 3
hours a day no matter what.

5) How many preptests did you do? Almost all available except the hard ones to find... probably about 50 twice
each.

6) What would you change if you were to do it again? Contemplated jumping off a cliff less while waiting for
my score.

7) Any other misc comments/suggestions. Do every prep test. If you are getting around a 160, you can study on
your own and see why you are getting things incorrect. Study reading comprehension A LOT. a lot of people
assume they're good at it, and all the courses seem to treat it as the least important but you can get 28/28 or
27/27 here if you prepare as much as people prepare for logic games.

Thanks,
Indy

ConsideringLawSchool

Hi everyone--

I used this board a lot when I was studying for the LSAT, and I definitely would not have done very well
without all the advice and tips from folks on here (thanks to everyone who patiently addressed all the individual
questions I posted). I was fortunate to get a 177 on my first attempt. I do not claim that the advice below is
original, but I have tried to collect the strategies that helped me the most as a way to give back for all the help I
received

I think that this approach may be particularly helpful to the following folks:
 People who score 160+ with limited prep and are trying to get 175+
 People who are very busy with work/school/life and have limited study time
 People who have a limited budget to spend on prep

Below are the materials that I would recommend using:


 The Official LSAT SuperPrep ($18)
 PowerScore LG Bible ($40.94)
 CambridgeLSAT Recent PrepTests (39-59) ($115)
 CambridgeLSAT Difficult LR Questions Bundle ($17) (http://www.cambridgelsat.com/productdet ...
stions/336)
 CambridgeLSAT LG by Type ($35)
http://www.cambridgelsat.com/productdetail/lsat/by_bundle/logic_games_by_type_(preptests_1-38)/358
 A large-faced watch that is very easy to reset during 10 second breaks between sections ($20-$25). I
recommend something like: http://www.sears.com/shc/s/p_10153_1260 ... 921x00003a

The total cost here, assuming that you have access to free printing on a campus, at a workplace, or elsewhere is
$250.

I also recommend taking advantage of the following free resources:


 AtlasLSAT Forums - fabulous for getting great explanations of tricky questions
(http://www.atlaslsat.com/forums/)
 Wiki containing old LSAT questions explained - (http://en.wikibooks.org/wiki/Past_LSAT_Explained)
 Atlas strategies for certain game types (just search on this forum--I don't want to repost their copyright
materials)

General advice for taking practice tests:


 Go to a location other than your home that is fairly similar to (or a bit more hectic than) a test center. I
alternated between the reading room at the local public library and the tables at a busy urban train
station.
 Set a countdown timer with a low beep for each section (if you have a smartphone, you probably
have one built in). Put this timer out of view (just upside down on your desk is fine). You won't be able
to see the proctor's timer during the real thing.
 For your reference during the test, use the same large-faced watch that you will use during the real
thing. Set it to 11:25 before each section. Use your watch as a guide the way you will during the real
thing, but time is up when your out-of-sight timer beeps.
 Always use a bubble sheet.
 Mark any question where you don't feel 90+% confident. These marks will be useful in reviewing later--
and in helping you to learn when your uncertainty suggests you are wrong and when it doesn't.
.
General advice for reviewing PTs:
 Carefully review each question that you got wrong or that you got right but marked as uncertain. Many
people like to write out why each wrong answer was wrong and why the right answer was right. I was a
bit too lazy to do this consistently, but I do think that it would be very helpful. Someone on here gave
the great advice that the ability to see why wrong answers are wrong is just as helpful as the ability to
see why the right answer is right. If you can effectively eliminate 4 wrong answers, you're all set.
.
 If you're not completely sure that you understand why the right answer is right and the other 4 are
wrong, look for posted explanations on TLS, on the Atlas forum, or on the Wiki. You can always post
the question here or on Atlas, and folks are generally very kind in explaining.
.
 Use a simple spreadsheet to track how you many you miss on each section on each test. Also keep track
of your raw and scaled scores. There are some much more complex tracking spreadsheets that have been
shared on here. I personally did not use one, but I imagine they might be very helpful to certain people.
.
 Keep track of every single question where you were incorrect or uncertain. I just kept all my PTs
together and highlighted in yellow each question that was wrong. You could also clip the questions. Do
whatever seems easiest to you to keep track, but you will definitely want to revisit these questions in the
future.

Here is the basic study plan that I would recommend. I think that you could follow this plan in as little as
2-3 months, but starting earlier is obviously better:

 Begin by reading the LG Bible cover to cover, doing all the practice problems. I do not personally
think that doing a PT before you begin the LG Bible is particularly helpful. If you want to get a baseline
score, go for it. As far as I'm concerned, though, you just risk beginning to get into bad habits with the
logic games (and getting frustrated with a low score since you won't know the basic strategies for the
games). If there are any questions that you don't understand, come on here to find explanations. Go
though each and every game until you can do it in under 8.5 minutes. You may find it helpful to make a
photocopy of the games before you begin so you can redo them as many times as needed.

I personally think the LG Bible is great for all game types other than Pure Sequencing/Relative Ordering
and In-and-Out games. For those two game types, I prefer the Atlas strategies. One of their teachers has
been kind enough to post their approach in the past. For copyright reasons, I don't want to repost his
links, but I'm sure you can find them if you search the forums. Their pure sequencing strategy involves a
tree format that I think is much easier than the PS Bible's complex series of arrows and lines. Their in-
and-out strategy involves two columns with all the variables that I find much simpler than the PS
method.
.
 Read through the written portion of SuperPrep. The first 100 pages (as I recall) offer a great analysis of
question types and how to approach each section. There are also some practice questions. I really like
this resource since it is written by the testmakers and is concise but effective. The reading is dense, but
it's worth it, in my opinion.
.
 After you have finished the LG bible and the SuperPrep reading material, take a practice test under
strictly timed conditions. If you live in a city where Kaplan offers free proctored tests, I'd recommend
taking the test there (just being in a room with other test-takers and a proctor with a precise timer is very
different from sitting in your living room with your cat on your lap and the chips and salsa nearby).
After you review this test, you will know what your strengths and weaknesses are. If you ran out of time
on sections (other than LG), that's not a huge problem at this point. This score is just a baseline of sorts.
.
 Do each PT from SuperPrep under untimed conditions. The goal here is to come to understand each
question and answer. This book offers the wonderful resource of explanations written by LSAC. I would
read every word of these explanations--whether you get the questions right or wrong. They really helped
me to appreciate what sorts of reasoning the testmakers have in mind. Go back and revisit questions as
many times as needed until you have them mastered.
.
 Depending upon how much time you have to study, your next goal is to master doing hard questions
effectively. Ideally, you would have a month to dedicate to this phase of your studying. I would next go
through the "Difficult Questions" LR set from CambridgeLSAT (again, this advice is primarily for
folks who are doing well on most LR questions and want to get into the mid-to-high 170s) and the Logic
Games from PTs 1-39. You may want to incorporate into this stage some RC sections too. I did not, and
I somewhat regretted that. There are RC sections by type on CambridgeLSAT, so you may wish to
download some sections that tend to be challenging for you.
.
o There are 400 questions in the LR "Difficult Questions" set. If you have a busy schedule, you
can just do these questions a few at a time when you have down time. I took them with me
wherever I went and did them whenever I was sitting on a bus, waiting for a friend, on a boring
conference call, or on my lunch break from work. Time doesn't matter for these questions (within
reason). All that matters is that you are able to figure out these hardest questions correctly. As
with the PTs, mark and set aside any questions that you got wrong.
.
o Many people will tell you that the LGs from PTs 1-39 are difficult and funky. They are. Once
you can master them, you will not be thrown by any LG that you encounter, even if they do not
fit in the PowerScore mold. If you are able to do so, I would do all of these LG sections under
timed conditions. Set aside 35 minutes whenever you can to do these sections. I did them
primarily on the train to and from work each day (so it took me about 3 weeks to get through all
of them). As with the PTs, mark and set aside any games you got wrong or could not complete
during the allotted time.
.
 You are now ready to tackle full tests under timed conditions. You want to dedicate at least the
final month of your prep to this objective.
o There are many people on here who will tell you that you should do every single preptest (1-59).
For those who are already doing well and have busy schedules, I recommend at least doing PTs
39-59. I decided to set aside 5:00 PM - 10:00 PM each weekday for studying during the final
week. During 4-5 weeks, do 4-5 preptests each week.
o Whether you do 4-section or 5-section preptests is up to you. If you do 5-section, add as the extra
section a section from PTs 1-39. Generally add whatever type of section is most difficult for you
to get the extra practice. If endurance is an issue for you, definitely do 5-section (or even 6). If
you don't feel endurance is a concern and you are very busy, you may prefer to stick with 4-
section. If you do 4-section, don't take any breaks between the sections (you'll build up
endurance--and each PT will only take 2 hrs, 20 minutes).
o Review each PT right after you finish it as described above.
.
 1-2 weeks before the test, go back and review every question you have marked as problematic since
you started studying. Redo these questions until you are getting them right.
.
 During the final week, do a couple new PTs. If you're going to be stressed out by the scores, don't score
them. (Correct the answer sheet, but don't calculate the score.)
.
 Do a few familiar practice questions on your way to the test.
.
 Relax, do your best, have a drink!

I hope this advice is helpful. Pick what might be useful, and throw out the rest. Good luck!

You might also like